Rząd liczb PDF Drukuj Email
Zadania II
Wpisany przez Joachim Jelisiejew   
niedziela, 07 lutego 2010 19:41

Zadania 
Zadania PDF.

Zadania 
Rozwiązania PDF.

Źródło zadań w texu.

 
\documentclass[10pt]{article}
\usepackage{amssymb}
\usepackage{amsmath}
\textwidth 16cm
\textheight 24cm
\oddsidemargin 0cm
\topmargin 0pt
\headheight 0pt
\headsep 0pt
\usepackage[polish]{babel}
\usepackage[utf8]{inputenc}
\usepackage[T1]{fontenc}
%\usepackage{MnSymbol}
% ----------------------------------------------------------------
\vfuzz4pt % Don't report over-full v-boxes if over-edge is small
\hfuzz4pt % Don't report over-full h-boxes if over-edge is small
% THEOREMS -------------------------------------------------------
\newtheorem{thm}{Twierdzenie}[section]
\newtheorem{cor}[thm]{Wniosek}
\newtheorem{lem}[thm]{Lemat}
\newtheorem{defn}[thm]{Definicja}
\newtheorem{tozs}[thm]{Tożsamość}
\newtheorem{hyp}[thm]{Hipoteza}
\newtheorem{useless}[thm]{}
 
\def\mb#1{\mathbb{#1}}
\def\rozw{\textbf{Rozwiązanie}: \\}
\def\deg{^{\circ}}
\def\source#1{$ $\\Źródło: #1}
\def\o{\operatorname{ord}}
 
\begin{document}
\title{II kółko -- Czy mamy dość teorii liczb?}
\date{27.10.09}
\maketitle
 
\paragraph{Teoria}
\begin{enumerate}
    \item Małe twierdzenie Fermata:
        \begin{thm}[Małe twierdzenie Fermata]
            Dla każdej liczby pierwszej $p$ i liczby całkowitej $a$ zachodzi
            $$p | a^p - a$$
        \end{thm}
    \item \begin{defn}
            Niech $a, n$ będą liczbami naturalnymi względnie pierwszymi.
            \emph{Rzędem} liczby $a$ modulo $n$ nazywamy najmniejsze
            $k\in\mb{Z}_+$, takie, że 
            $$a^k \equiv 1 \mod n$$
            Rząd ten oznaczamy przez $\o(a, n)$ lub, gdy $n$ jest znane,
            $\o(a)$.
        \end{defn}
    \item \begin{thm}[Lemat o rzędzie]
            Jeżeli $a, k', n$ są takie, że
            $$a^{k'} \equiv 1 \mod n$$
            to $\displaystyle{\o(a, n) | k'}$.
        \end{thm}
    \item \begin{cor}
            Jeżeli $p$ jest liczbą pierwszą, zaś $a$ jest liczbą całkowitą
            niepodzielną przez $p$, to
            $$\o(a, p)|p-1$$
        \end{cor}
\end{enumerate}
 
\paragraph{Zadania}
\begin{enumerate}
    \item Udowodnić, że dla liczby pierwszej $p$ istnieje takie $n$ naturalne, że
        $$p|2^n - n$$
        \source{Staszic}
    \item Udowodnić, że dla liczby pierwszej $p$ istnieje nieskończenie wiele
        liczb naturalnych $n$ takich, że
        $$p|2^n - n$$
        \source{Staszic}
    \item Rozstrzygnąć, dla jakich $k\in \mb{N}$ istnieje liczba naturalna
        $n$, będąca kwadratem liczby naturalnej i zaczynająca się w zapisie
        dziesiętnym $k$ jedynkami.
        \source{Mathlinks}
    \item * Udowodnić, że równanie
        $$x^2 + y^2 + z^2 = (x-y)(y-z)(z-x)$$
        ma nieskończenie wiele rozwiązań w liczbach naturalnych $x,y,z$.
        \source{Mathlinks}
    \item * Znajdź wszystkie liczby naturalne $n$ takie, że
        $$n^2 | 3^n + 1$$
        \source{Staszic}
\end{enumerate}
 
\paragraph{Zadania ,,lekcyjne''}
\begin{enumerate}
    \item Udowodnij, że dla każdej liczby naturalnej $n$ zachodzi
        $$2^{n+1} | 3^{2^n} - 1$$
    \item Rozstrzygnąć, czy poniższe twierdzenie jest prawdziwe:\\
        Jeżeli $p>2$ jest liczbą pierwszą, zaś
        $a$ jest taką liczbą całkowitą dodatnią, że $p | a+1$, to
        $$p^{n+1} | a^{p^n} + 1$$
        dla wszystkich liczb $n\in \mb{Z}_+$.
\end{enumerate}
 
\end{document}
 
 
 
 

Źródło rozwiązań w texu.

 
\documentclass[10pt]{article}
\usepackage{amssymb}
\usepackage{amsmath}
\textwidth 16cm
\textheight 24cm
\oddsidemargin 0cm
\topmargin 0pt
\headheight 0pt
\headsep 0pt
\usepackage[polish]{babel}
\usepackage[utf8]{inputenc}
\usepackage[T1]{fontenc}
%\usepackage{MnSymbol}
% ----------------------------------------------------------------
\vfuzz4pt % Don't report over-full v-boxes if over-edge is small
\hfuzz4pt % Don't report over-full h-boxes if over-edge is small
% THEOREMS -------------------------------------------------------
\newtheorem{thm}{Twierdzenie}[section]
\newtheorem{cor}[thm]{Wniosek}
\newtheorem{lem}[thm]{Lemat}
\newtheorem{defn}[thm]{Definicja}
\newtheorem{tozs}[thm]{Tożsamość}
\newtheorem{hyp}[thm]{Hipoteza}
\newtheorem{useless}[thm]{}
 
\def\mb#1{\mathbb{#1}}
\def\rozw{\textbf{Rozwiązanie}: \\}
\def\deg{^{\circ}}
\def\source#1{$ $\\Źródło: #1}
\def\o{\operatorname{ord}}
 
\begin{document}
\def\comment#1{\ [[#1]]\ }
\title{``Czy mamy dość teorii liczb?'' - rozwiązania}
\author{Mateusz Jocz}
\date{}
\maketitle
 
\begin{enumerate}
    \item Udowodnić, że dla liczby pierwszej $p$ istnieje takie $n$ naturalne, że
        $$p|2^n - n$$
        \source{Staszic}
    \item Udowodnić, że dla liczby pierwszej $p$ istnieje nieskończenie wiele
        liczb naturalnych $n$ takich, że
        $$p|2^n - n$$
        \source{Staszic}
 
        \textbf{Rozwiązanie:}
    \begin{enumerate} 
        \item Jeżeli $p=2$ to teza jest spełniona dla $n=2$. Dalej zakładamy że
    $p\neq2$. 
        \item Z małego twierdzenia Fermata mamy:
    \[2^{p-1} \equiv 1 \mod p\]
    Teraz na mocy \textit{lematu o rzędzie} otrzymujemy $\o(2,p)|p-1$.
    Z tego wynika, że liczby $\o(2,p)$ i $p$ są względnie pierwsze.
        \item Wykażemy, że teza jest spełniona dla pewnego $n=kd$, gdzie $k$ jest
    całkowite, a $d=\o(2,p)$. Dla każdego $k$, z definicji rzędu $d$, mamy:
    \[(2^{d})^{k} \equiv 1^{k}\mod p\]
    \[2^{n} \equiv 1\mod p\]
        \item Rozważmy zbiór liczb $A=\{0,d,2d,\ldots,(p-1)d\}$. Wiemy, że $p$ jest
    względnie pierwsze z $d$. Jeżeli istniałyby dwie liczby $h$ i $l$ $(0\leq
    l<h<p)$, takie że $hd$ i $ld$  dawałyby takie same reszty z dzielenia
    przez $p$ to mielibyśmy $p|d(h-l) \Rightarrow p|h-l$, sprzeczność. Zatem
    liczby ze zbioru $A$ dają różne reszty z dzielenia przez $p$, a skoro w
    $A$ jest tyle samo liczb, co w zbiorze wszystkich reszt, to liczby z $A$ dają wszystkie reszty
    z dzielenia przez $p$, w szczególności dla pewnego $m\in\{0,1,2,\ldots,p-1\}$ mamy:
    \[md \equiv 1 \mod p\]
        \item Definiujemy 
            $$n:=md$$
            gdzie $m,d$ określone są wyżej. Udowodniliśmy, że
            $$2^n \equiv 1 \mod p \hbox{ oraz } n\equiv 1\mod p$$
            Stąd wynika, że
            $$p | 2^n - n$$
            Istnieje więc liczba $n$ spełniająca warunki zadania, pozostaje
            dowieść, że takich liczb jest nieskończenie wiele.
        \item Niech $z$ będzie dowolną liczbą naturalną, $d=\o(2,p)$, jak
            wyżej, a $n$ będzie takie, że $n \equiv 2^n\equiv 1 \mod p$.  Zauważmy że: 
    \[n+zpd \equiv n \equiv 1 \mod p\]
    \[2^{n+zpd} = 2^n \cdot (2^{d})^{zp} \equiv 1\cdot 1^{zp} = 1 \mod p\]
    \[2^{n+zpd} - (n + zpd) \equiv 0\mod p \]
    Ponieważ $z$ może być dowolną liczbą naturalną to otrzymujemy
    nieskończenie wiele rozwiązań postaci $n + zpd$.
    \end{enumerate}
 
    \item Rozstrzygnąć, dla jakich $k\in \mb{N}$ istnieje liczba naturalna
        $n$, będąca kwadratem liczby naturalnej i zaczynająca się w zapisie
        dziesiętnym $k$ jedynkami.
        \source{Mathlinks}
 
        \textbf{Rozwiązanie:}
        \begin{enumerate}
        \item Dla $k=0$ taka liczba oczywiście istnieje, na przykład
            $n=4=2^{2}$.
        \item Dla $k\geq 1$ mamy:
        \[{\underbrace{33\ldots 3}_{k-1\ razy}4}^{2}=(\frac{10^{k-1}-1}{3}\cdot 10 +4)^{2}=\]
        \[=\frac{10^{2k-2}-2\cdot10^{k-1}+1}{9}\cdot 10^{2}+8\cdot10\cdot \frac{10^{k-1}-1}{3}+16=\]
        \[=\frac{10^{2k}}{9}- 20\cdot\frac{10^k}{9}+\frac{100}{9}+24\cdot \frac{10^{k}}{9}-\frac{240}{9}+\frac{90}{9}+6=\]
        \[=\frac{10^{2k}}{9}-\frac{10^{k}}{9}+5\cdot \frac{10^{k}}{9}-\frac{50}{9}+6=\]
        \[=\frac{10^{k}-1}{9} \cdot 10^{k}+5 \cdot \frac{10^{k-1}-1}{9}\cdot 10 +6=\]
        \[=\underbrace{11\ldots1}_{k\ razy}\underbrace{55\ldots5}_{k-1\ razy}6\]
        Stąd już widzimy, że o ile $k\geq 1$ to taka liczba $n$ dla każdego $k$ istnieje. 
        \item Ostatecznie liczba naturalna $n$ o wymaganych własnościach istnieje dla wszystkich $k\in \mb{N}$,
            \newline $c.k.d.$
%        \item Ogólnie prawdziwa jest równość:
%        \[({\underbrace{33\ldots 3}_{k+1\ razy}\cdot10^{[\log{\alpha}]+1}+\alpha})^{2}=
%        \underbrace{11\ldots1}_{k\ razy}\cdot10^{[\log{\beta}]+1}+\beta\]
%        dla każdego naturalnego $\alpha$ i pewnego naturalnego $\beta$.
%        Dla przykładu:
            \item Oczywiście istnieją też inne przykłady podobnych liczb, np.:
        \[{\underbrace{33\ldots 3}_{k+1\ razy}}^{2}=\underbrace{11\ldots1}_{k\ razy}0\underbrace{88\ldots8}_{k\ razy}9\]
        \[{\underbrace{33\ldots 3}_{k-1\ razy}45}^{2}=\underbrace{11\ldots1}_{k\ razy}\underbrace{88\ldots8}_{k-2\ razy}9025\]
            \item \textbf{Komentarz:} Takie rozwiązanie narzuca się, jeżeli
                pomyśli się, że liczba $111111\dots$ jest prawie równa
                $10^{2k} \cdot 1/9$, a przecież ta liczba jest kwadratem
                liczby $10^k \cdot 1/3$ równej $333333\dots$. Potem te
                ``prawie'' trzeba oczywiście sformalizować...
        \end{enumerate}
 
    \item * Udowodnić, że równanie
        $$x^2 + y^2 + z^2 = (x-y)(y-z)(z-x)$$
        ma nieskończenie wiele rozwiązań w liczbach naturalnych $x,y,z$.
        \source{Mathlinks}
 
        \textbf{Rozwiązanie:}
        \begin{enumerate}   
 
            \item Niech $k$ będzie dowolną liczbą całkowitą dodatnią. Rozważmy rozwiązania postaci: 
            \[ (x,y,z)=(\ (2k-1)(6k^{2}+1), 2k(6k^{2}+1), (2k+1)(6k^{2}+1)\ )\]
                Obliczamy, że lewa strona równania jest równa:
                \[ x^{2}+y^{2}+z^{2}=(2k-1)^{2}(6k^{2}+1)^{2}+(2k)^{2}(6k^{2}+1)^{2} + 
                (2k+1)^{2}(6k^{2}+1)^{2}=\]
                \[=(6k^{2}+1)^{2}(4k^{2}-4k+1+4k^{2}+4k^{2}+4k+1)=(6k^{2}+1)^{2}(12k^{2}+2)=2(6k^{2}+1)^{3}\]
                Natomiast prawa strona równania wynosi:
                \[(x-y)(y-z)(z-x)=(6k^{2}+1)(2k-1-2k)\cdot(6k^{2}+1)(2k-2k-1)\cdot(6k^{2}+1)(2k+1-2k+1)=\]
                \[=(6k^{2}+1)^{2}\cdot 2(6k^{2}+1)= 2(6k^{2}+1)^{3}\]
                Widzimy, że obie strony równania są równe. Ponieważ $k$ jest
                dowolną liczbą całkowitą dodatnią i dla różnych $k$ wartości
                liczby $z=(2k+1)(6k^2 + 1)$ są różne, to otrzymaliśmy nieskończoną 
                liczbę rozwiązań postaci 
                \[ (x,y,z) = (\ (2k-1)(6k^{2}+1), 2k(6k^{2}+1),
                (2k+1)(6k^{2}+1)\ ) \]
                \textbf{Koniec dowodu -- punkt następny służy tylko
                objaśnieniu i nie potrzeba go pisać np. na olimpiadzie.}
            \item \emph{W jaki sposób możemy znaleźć właśnie taką postać
                rozwiązania?}\\
                \begin{enumerate}
                    \item Na początku stwierdzamy, że trzy niewiadome to trochę za dużo. Spróbujmy ograniczyć się do dwóch.
                 Warto więc aby $(x,y,z)$ był jakimś ciągiem, którego wyrazy możemy zapisać za pomocą dwóch niewiadomych.
                 Jako jeden z pierwszych nasuwa nam się ciąg arytmetyczny. Różnicę naszego ciągu oznaczmy przez $r$. 
                 Prawdziwe są równości: 
                 $$x=y-r\ \ \hbox{ i }\ \ z=y+r$$
                 Po podstawieniu tych równości możemy zredukować równanie
                        $$x^2 + y^2 + z^2 = (x-y)(y-z)(z-x)$$
                        do postaci:
                 $$3y^{2}=2r^{2}(r-1)$$
               \item Z powyższego równania możemy wyliczyć $y$:
                        $$y = \sqrt{\frac{2r^2(r-1)}{3}} =
                        r\sqrt{\frac{2(r-1)}{3}}$$
                        Jeżeli teraz znajdziemy nieskończenie wiele takich $r$
                        całkowitych dodatnich, że
                        $\displaystyle{r\sqrt{\frac{2(r-1)}{3}}}$ jest
                        całkowite (oczywiście jest też wtedy dodatnie) i przyjmiemy
                        $$r_y:=r\sqrt{\frac{2(r-1)}{3}},\ r_x:=r_y - r,\
                        r_z:=r_y + r$$
                        to trójka $(r_x,r_y,r_z)$ będzie rozwiązaniem naszego
                        wyjściowego równania.
                    \item Kiedy $\sqrt{\frac{2(r-1)}{3}}$ może być
                        całkowite?\\
                        Żeby było to całkowite, wystarczy, żeby $r$ było
                        postaci $r=6k^2+1$, gdzie $k\in \mathbb{Z}_+$ -- wtedy
                        $\sqrt{\frac{2(r-1)}{3}} = 2k$.\\
                        Przyjmujemy więc takie $r$ i wyliczamy
                        $$r_y = 2k(6k^2 + 1),\ r_x = (2k-1)(6k^2+1),\ r_z =
                        (2k+1)(6k^2 +1)$$
%                 $$3y^{2}-2r^{2}(r-1)=0$$
%                 Wiedząc że $$y=\frac{\pm\sqrt{\Delta}}{6}\hbox{,}$$ $$\hbox{gdzie}\  \Delta \hbox{ jest wyróżnikiem
%                 rozważanego równania}$$
%                 chcemy by delta była kwadratem liczby całkowitej i ponadto była podzielna przez 36.
%                 $$\Delta=24r^{2}(r-1)=4\cdot r^{2}\cdot6(r-1)$$
%                 Wyrażenia $4$ i $r^{2}$ nas mało obchodzą bo są kwadratami liczb całkowitych. Nas obchodzi tylko to, 
%                 aby wyrażenie $6(r-1)$ było kwadratem liczby całkowitej. Więc bierzemy $r-1=6k^{2}$, gdzie $k$ jest 
%                 całkowite. Przy okazji okazuje się, że teraz delta jest faktycznie podzielna przez 36. 
%                 $$r=6k^{2}+1$$
%                 $$\Delta=4\cdot (6k^{2}+1)^{2}\cdot 36k^{2}$$
%                 Interesuje nas dodatnia wartość y, więc:
%                 $$y=\frac{\sqrt{4\cdot (6k^{2}+1)^{2}\cdot 36k^{2}}}{6}=
%                        \frac{2\cdot (6k^{2}+1)\cdot 6k}{6}=2k(6k^{2}+1)$$
%                 Pozostaje nam wyliczyć $x$ oraz $z$, pamiętając że:
%                 $$x=y-r\ \ \hbox{ i }\ \ z=y+r$$
%                 \[ x=(2k-1)(6k^{2}+1)\ \ \hbox{ i }\ \ z=(2k+1)(6k^{2}+1)\] 
                \end{enumerate}
            \end{enumerate}
 
    \item * Znajdź wszystkie liczby naturalne $n$ takie, że
        $$n^2 | 3^n + 1$$
        \source{Staszic}
 
        \textbf{Rozwiązanie:}
        \begin{enumerate}   
            \item Załóżmy, że dla pewnego $n>1$ teza jest
                spełniona. Wtedy możemy $n$ przedstawić w postaci: $n=k\cdot
                p$, gdzie $p$ jest \textbf{najmniejszym} pierwszym dzielnikiem liczby
                $n$, a $k$ jest pewną liczbą całkowitą dodatnią.
            \item
                Zauważmy, że $3\not|n$ i tym samym $p\neq 3$. Sensowny jest więc napis $\o(3,p)$
                Zauważmy, że:
                \[ 3^{n} \equiv -1 \mod p \]
                \[ 3^{2n} \equiv 1 \mod p \]
                Na mocy \textit{lematu o rzędzie} mamy $\o(3,p) | 2n = 2pk$.
                Z drugiej strony z (wniosku z) \textit{lematu o rzędzie} wynika że $\o(3,p)|p-1$, 
                czyl $\o(3,p)$ jest względnie pierwsze z $p$. Zatem $\o(3,p) | 2k$. 
            \item Niech $d$ będzie największym wspólnym dzielnikiem liczb
                $\o(3,p)$ i $k$.\\
                Możliwe są dwa przypadki: $\o(3,p)=d$ lub $\o(3,p)=2d$.
                W obu przypadkach, jeżeli $d\geq p$, to $\o(3,p)\geq p$ i 
                otrzymujemy sprzeczność z $\o(3,p)|p-1$. Stąd wynika $d<p$.\\
            \item 
                Liczby $d<p$ i $p$ są dzielnikami $n$, przy czym $p$
                jest najmniejszym dzielnikiem pierwszym, więc $d=1$ (kluczowy
                moment) i tym samym
                $$\o(3,p)=1 \hbox{ lub } \o(3,p)=2$$
                korzystając z definicji rzędu otrzymujemy
                \[3^{1} \equiv 1 \mod p \hbox{ lub } 3^{2} \equiv 1 \mod p\]
                \[p|2 \hbox{ lub } p|8\]
                A więc $p=2$, $n=2k$, czyli $4|n^{2}$. Zauważmy, że jeśli $n$ jest parzyste to
                $3^{n} \equiv  1 \mod 4$. Liczba $3^{n} +1$ nie może więc być
                podzielna przez $4$. Sprzeczność.\\
            \item Pozostaje nam przypadek gdy $n=1$. Łatwo zauważyć, że teza jest teraz
                spełniona. Jest to więc jedyne rozwiązanie.\end{enumerate}
 
        \end{enumerate}
 
%\end{enumerate}
 
\end{document}